Difference between revisions of "1994 AJHSME Problems/Problem 1"
(Created page with "==Problem== Which of the following is the largest? <math>\text{(A)}\ \dfrac{1}{3} \qquad \text{(B)}\ \dfrac{1}{4} \qquad \text{(C)}\ \dfrac{3}{8} \qquad \text{(D)}\ \dfrac{5}{12...") |
MathsSense (talk | contribs) |
||
Line 3: | Line 3: | ||
<math>\text{(A)}\ \dfrac{1}{3} \qquad \text{(B)}\ \dfrac{1}{4} \qquad \text{(C)}\ \dfrac{3}{8} \qquad \text{(D)}\ \dfrac{5}{12} \qquad \text{(E)}\ \dfrac{7}{24}</math> | <math>\text{(A)}\ \dfrac{1}{3} \qquad \text{(B)}\ \dfrac{1}{4} \qquad \text{(C)}\ \dfrac{3}{8} \qquad \text{(D)}\ \dfrac{5}{12} \qquad \text{(E)}\ \dfrac{7}{24}</math> | ||
+ | |||
+ | ==Solution== | ||
+ | |||
+ | <math>\dfrac{3}{8}</math> and <math>\dfrac{5}{12}</math> are the only fractions where the numerator is close to half of the denominator. <math>\dfrac{3}{8}</math> is equal to .375 and <math>\dfrac{5}{12}</math> is equal to approximately .41. <math>\boxed{\text{(D)}}</math> |
Revision as of 16:51, 14 November 2011
Problem
Which of the following is the largest?
Solution
and are the only fractions where the numerator is close to half of the denominator. is equal to .375 and is equal to approximately .41.